0 Daumen
227 Aufrufe

Untersuchen Sie das System von Differenzengleichungen \( \boldsymbol{x}_{k+1}=\mathbf{A} \boldsymbol{x}_{k}, k=0,1,2,3, \dots \)

 auf asymptotische Stabilität, wobei


\( \mathbf{A}=\left(\begin{array}{lll}0 & 4 & 0 \\ 0 & 2 & 2 \\ 0 & 2 & 4\end{array}\right) \)


wie muss ich hier vorgehen?

mfg

Avatar von

Ein anderes Problem?

Stell deine Frage

Willkommen bei der Mathelounge! Stell deine Frage einfach und kostenlos

x
Made by a lovely community